If f(x) = x2 + 7x and g(x) = 3x - 1, what is f(g(x))?

Answers

Answer 1

Answer:

f(g(x)) = 9x^2 + 15x - 6

Step-by-step explanation:

We are using function g(x) = 3x - 1 as the input to function f(x) = x^2 + 7x.

Starting with f(x) = x^2 + 7x, substitute g(x) for x on the left side and likewise substitute x^2 + 7x for each x on the right side.  We obtain:

f(g(x)) = (3x - 1)^2 + 7(3x - 1).

If we multiply this out, we get:

f(g(x)) = 9x^2 - 6x + 1 + 21x - 7, or

f(g(x)) = 9x^2 + 15x - 6


Related Questions

Cheyenne's bi-weekly gross pay is $529.81. She sees that $18.54 was deducted for Medicare tax. What percent of Cheyenne's gross pay has been withheld for Medicare tax? Round to the nearest tenth. (2 points)
0.35%
2.2%
3.5%

Answers

18.54/529.81= 0.0349
0.0349*100=3.49%= 3.5%

Answer:

I got 3.5%

Step-by-step explanation:

Hope this helps

Solve for the value of x

Answers

Answer:

Step-by-step explanation:

This is the Law of Cosines. Use the following formula:

[tex]5.1^2=3.3^2+3.3^2-2(3.3)(3.3)cos(x)[/tex] and simplify to

26.01 = 10.89 + 10.89 - 21.78cos(x) and a bit more to

4.23 = -21.78cos(x) and finally to

-.194214876 = cos(x) and use the 2nd button along with the cos button to find that the missing angle is 101.2 degrees

Chen rode his skateboard 3/3/4 miles in 34 of an hour.

What was his average speed in miles per hour?

_[blank]_ miles per hour

Answers

Answer:

5 miles per hour

Step-by-step explanation:

3¾ ÷ ¾ =

15/4 ÷ ¾ =

15/4 x 4/3 =

15/3 = 5

N is one of the numbers below. N is such that when multiplied by 0.75 gives 1. Which number is equal to
N?
A) 1 1/2
B 1 1/3
C) 5/3
D) 3/2

Answers

Answer:

it should be letter c 5/3 I could be wrong but I hope this help

The function f(x) = −x2 + 18x − 72 models the daily profit, in dollars, a gym makes for selling memberships, where x is the number of memberships sold, and f(x) is the amount of profit.

Part A: Determine the vertex. What does this calculation mean in the context of the problem? (5 points)

Part B: Determine the x-intercepts. What do these values mean in the context of the problem? (5 points)

(10 points)

Answers

Answer:

Step-by-step explanation:

The way to do this so as to streamline both the vertex and finding the zeros is to complete the square. That method will provide us with the vertex, and then we can continue on to factor from that form to find the zeros. Completing the square requires us to set the quadratic equal to 0 then move over the constant, giving us

[tex]-x^2+18x=72[/tex] The leading coefficient HAS to be a positive 1; ours is negative 1 so we factor out the negative to get:

[tex]-(x^2-18x)=72[/tex] Now we're ready to complete the square.

Take half the linear term, square it, and add it to both sides. Our linear term is 18 (from -18x; don't worry about the negative because squaring it makes it positive anyway). Half of 18 is 9, and 9 squared is 81.

BUT on the left we have that -1 sitting out front that refuses to be ignored. What we actually added on to the left side, inside the parenthesis, is -1(81) which is -81.   -81 is what we add to the right since that turns out to be what we added to the left:

[tex]-(x^2-18x+81)=72-81[/tex] and we clean that up.

The reason we complete the square is because when we simplify the left side, we end up with a perfect square binomial found from taking the square root of x-squared, the first sign we come to, then the square root of 81:

[tex]-(x-9)^2=-9[/tex].  Move the constant back over to get

[tex]-(x-9)^2+9=y[/tex] telling us that the vertex is (9, 9). In the context of the problem that means that the gym sells on average 9 memberships a day and the profit it makes on average per day is $9.

To factor, we will go back one step to

[tex]-(x-9)^2=-9[/tex] and begin by dividing both sides by -1 to get

[tex](x-9)^2=9[/tex] and undo the squaring by taking the square root of both sides to get

x - 9 = ±3 so

x = 9 + 3 and

x = 9 - 3 so

x = 6 and 12

Those are the zeros. This means that if they sell either 6 or 12 memberships they have a 0 profit. That may sound strange, but in business it does often work like that...selling too many of something makes your company lose money (this is often due to the cost required by you to produce or manufacture the product).

Annie is opening a savings account which earns 5.2% interest compounded continuously how much will she need to deposit in the account so she has $2300 after seven years

Answers

Hi

let's call X the initial deposit.

the interest rate is 5.2 %

so each year X increase by 1.052.

so we have : X *1.052^7 =2300

X = 2300/1.052^7

X = 1612,94

please note that the deposit was rounded to the next cent. as the result would be 1612,937...

Answer:

1686.22

Step-by-step explanation:

1686.22

 2300=P(1+(0.052x7))  

2300=P1.364  

P=2300/1.364  

=1686.217...  

=1686.22..

=1686 for the nearest dollar

what is 2x2x2x3x3 please give me answer

Answers

Answer:

The answer is 72.

I am right .

Vas happenin!
Hope your day is good

2 time 2 is 4
4 times 2 is 8
8 times 2 is 16
16 times 3 is 48
48 times 3 is 144
144 is your answer
Hope this helps *smiles*

In a standardized normal distribution the mean is ____ while the standard deviation is ____.
A. 0; 1
B. 1; 0
C. 0; 0
D. 1; 1

Answers

Answer:

A. 0; 1

Step-by-step explanation:

Required

Mean and standard deviation of a standardized normal distribution

A standardized normal distribution is represented as:

[tex](\mu,\sigma) = (0,1)[/tex]

This implies that:

[tex]\mu = 0[/tex] -- mean

[tex]\sigma = 1[/tex] --- standard deviation

Hence, (a) is true

Ronald types 360 words in 9 minutes.

If he types at a constant rate, how many words does Ronald type in 1 minute?

Answers

answer would be 40 words per min
360/9=40

Answer:

40 per minute.

Step-by-step explanation:

360/9=40

What is the perimeter of the triangle?
units

Answers

Answer:

Does the answer help you?

Find y when x = 22, if y varies directly as x,
and y = 42 when x = 5.

Answers

Answer:

184.8

Step-by-step explanation:

y =kx

k=y/x

k=42/5=8.4

y=8.4*22

Multiplying and bmbFREE BRAINLIST AND POINTS!! Fast! RIGHT ANSWERS ONLY! Scam and wrong answers will be reported and dealed with.
1. 6x(-4)=
3. (-11) x 5=
4. (-12) x (-7)=
5. (-2) x (-10)
6. 4 x (-15)=

Answers

Answer:

Step-by-step explanation:

-24

-55

84

20

-60

Instructions: Determine if the two triangles in
the image are congruent. If they are, state how
you know by identifying the postulate.
th

Answers

The 2 triangles are congruent

What is the slope of the points (-2,7) and (2,-5)?
4
-3
-12
3

Answers

Answer:

-3

Step-by-step explanation:

Slope is equal to (-5-7)/(2-(-2)=-12/4=-3

Functions, f and g are given by f(x)= 3+ cos x and g(x) = 2x, x is a real number. Determine the value of c for which f(g(x))= g(f(x)) where 0[tex]\leq[/tex] x<2[tex]\pi[/tex]

Answers

9514 1404 393

Answer:

  x = π

Step-by-step explanation:

You want f(g(x)) = g(f(x)):

  3 +cos(2x) = 2(3 +cos(x))

  cos(2x) -2cos(x) = 3 . . . . . . . rearrange

  2cos(x)²-1 -2cos(x) = 3 . . . . . use an identity for cos(2x)

  2(c² -c -2) = 0 . . . . . . . . . . . . substitute c = cos(x)

  (c -2)(c +1) = 0 . . . . . . . . . . . factor

  c = 2 (not possible)

  c = -1 = cos(x) . . . . . true for x = π

The value of x that makes f(g(x)) = g(f(x)) is x = π.

_____

Additional comment

The substitution c=cos(x) just makes the equation easier to write and the form of it easier to see. There is really no other reason for making any sort of substitution. In the end, the equation is quadratic in cos(x), so is solved by any of the usual methods of solving quadratics.

Independent simple random samples are selected to test the difference between the means of two populations whose standard deviations are not known. We are unwilling to assume that the population variances are equal. The sample sizes are n1 = 25 and n2 = 35. The correct distribution to use is the:
1) t distribution with 59 degrees of freedom.
2) t distribution with 58 degrees of freedom.
3) t distribution with 61 degrees of freedom.
4) t distribution with 60 degrees of freedom.

Answers

Answer:

2) t distribution with 58 degrees of freedom.

Step-by-step explanation:

Population standard deviations not known:

This means that the t-distribution is used to solve this question.

The sample sizes are n1 = 25 and n2 = 35.

The number of degrees of freedom is the sum of the sample sizes subtracted by the number of samples, in this case 2. So

25 + 35 - 2 = 58 df.

Thus the correct answer is given by option 2.

properties of exponents. the answer is 1/2^3 i need help with the work

Answers

(2^-1)^2/2×2^0

2^(-1×2)/2^1

2^-2/2^1

2^(-2-1)

2^(-3)

(1/2)^3

Properties used (m^n)^a = m^na

(m)^-n = (1/m)^n

m^0 = 1

m^n/m^a = m^(n-a)

Must click thanks and mark brainliest

Find the missing length indicated

Answers

Answer:

60

Step-by-step explanation:

Use similar triangles or the ratios from the right triangle altitude theorem.

x/36 = (64 + 36)/x

x² = 3600

x = 60


Match each polynomial on the left with its two factors on the right.

Answers

Answer:

Hello

Step-by-step explanation:

[tex]Formula: \\\\\boxed{\Large a^3\pm b^3=(a \pm b)(a^2 \mp ab+b^2)}\\\\8x^3+1=(2x)^3+1^3=(2x+1)(4x^2-2x+1)\\\\8x^3-1=(2x)^3-1^3=(2x-1)(4x^2+2x+1)\\[/tex]

The factors of the expression 8x³ + 1 and 8x³ - 1³ will be (2x + 1) & (4x² – 2x + 1) and (2x – 1) & (4x² + 2x + 1), respectively.

What is a polynomial?

A polynomial expression is an algebraic expression with variables and coefficients. Unknown variables are what they're termed. We can use addition, subtraction, and other mathematical operations. However, a variable is not divisible.

The expression is given below.

8x³ + 1 and 8x³ - 1³

(2x)³ + 1³ and (2x)³ - 1³

We know that the formula is given as,

a³ + b³ = (a + b) (a² – ab + b²)

a³ – b³ = (a – b) (a² + ab + b²)

Then the expression is written as,

(2x)³ + 1³ = (2x + 1) [(2x)² – 2x + 1²]

(2x)³ + 1³ = (2x + 1) (4x² – 2x + 1)

(2x)³ – 1³ = (2x – 1) [(2x)² + 2x + 1²]

(2x)³ – 1³ = (2x – 1) (4x² + 2x + 1)

The factors of the expression 8x³ + 1 and 8x³ - 1³ will be (2x + 1) & (4x² – 2x + 1) and (2x – 1) & (4x² + 2x + 1), respectively.

More about the polynomial link is given below.

https://brainly.com/question/17822016

#SPJ2

A pizza parlor has a choice of 10 toppings for its pizzas. From these 10 toppings, how many different 7-topping pizzas are possible?

Answers

Answer:

120

Step-by-step explanation:

There are 10 possible toppings to choose from, you choose 7.

Using combinatorics, it's 10!/(7! 3!), or 120.

The formula is (total amount to choose from )! divided by (amount you choose)!(amount you don't choose)!

Or search up combination formula

A combination is an arrangement of a set of numbers from a total set where the order of the set is not relevant.

The formula for combination.

= [tex]^nC_r[/tex]

= n! / r! (n -r)!

The number of possible 7-toppings for the pizza is 120.

What is a combination?

A combination is an arrangement of a set of numbers from a total set where the order of the set is not relevant.

We have,

The total number of toppings = 10.

n = 10

The number of required toppings = 7.

r = 7

The formula for combination.

= [tex]^nC_r[/tex]

= n! / r! (n -r)!

The possible number of possible 7-toppings pizzas.

= [tex]^{10}C_7[/tex]

= (10 x 9 x 8) / (3 x 2)

= 120

Thus,

n = 10 and r = 7

[tex]^{10}C_7[/tex] = 120

The number of possible 7-toppings for the pizza is 120.

Learn more about combination here:

https://brainly.com/question/14355408

#SPJ2

By Using 0,2,4,5,6 Write The Smallest Number And the Greatest Number​

Answers

Answer:

smallest is 0 and greatest is 6

simple

Answer:

0 and 6

Step-by-step explanation:

Because 0 is means nothing.And the highest number is 6

The median age (in years) of the U.S. population over the decades from 1960 through 2010 is given by
f(t) = −0.2176t3 + 1.962t2 − 2.833t + 29.4 (0 ≤ t ≤ 5)
where t is measured in decades, with t = 0 corresponding to 1960.
(a) What was the median age of the population in the year 1970?
(b) At what rate was the median age of the population changing in the year 1970?
(c) Calculate f ''(1).

Answers

Considering the given function, we have that:

a) 28.31 years.

b) 0.3382 years a decade.

c) 2.6184.

What is the function?

The median age of the U.S. population in t decades after 1960 is:

f(t) = -0.2176t³ + 1.962t² - 2.833t + 29.4.

1970 is one decade after 1960, hence the median was:

f(1) = -0.2176 x 1³ + 1.962 x 1² - 2.833 x 1 + 29.4 = 28.31 years.

The rate of change was is the derivative when t = 1, hence:

f'(t) = -0.6528t² + 3.924t - 2.933

f'(1) = -0.6528 x 1² + 3.924 x 1 - 2.933 = 0.3382 years a decade.

The second derivative is:

f''(t) = -1.3056t + 3.924

Hence:

f''(1) = -1.3056 x 1 + 3.924 = 2.6184.

More can be learned about functions at https://brainly.com/question/25537936

#SPJ1

Given the arc, name the central angle.
FG
A. ∠GQJ
B. ∠FQG
C. ∠GQI
D. ∠HQI

Answers

Answer:

B

Step-by-step explanation:

Given arc FG then the central angle is the angle at the centre subtended by FG , that is

central angle = ∠ FQG

The central angle is B i.e ∠FQG

What is central angle?

A central angle exists an angle whose vertex stands present at the center of a circle created by the two radii as the sides of the angle.

In Mathematics, an “arc” exists as a smooth curve joining two endpoints. In general, an arc exists one of the portions of a circle. It is essentially a part of the circumference of a circle. Arc exists as a part of a curve. An arc can be a portion of some other curved constitutions like an ellipse but mostly guides to a circle.

The angle substended by the arc would be ∠FQG.

To learn more about  central angle refer to:

https://brainly.com/question/14205090

#SPJ2

About 12.5% of restaurant bills are incorrect. If 200 bills are selected at ran- dom, find the probability that at least 22 will contain an error. Is this likely or unlikely to occur

Answers

Answer:

0.7734 = 77.34% probability that at least 22 will contain an error. Probability above 50%, which means that this is likely to occur.

Step-by-step explanation:

Binomial probability distribution

Probability of exactly x successes on n repeated trials, with p probability.

Can be approximated to a normal distribution, using the expected value and the standard deviation.

The expected value of the binomial distribution is:

[tex]E(X) = np[/tex]

The standard deviation of the binomial distribution is:

[tex]\sqrt{V(X)} = \sqrt{np(1-p)}[/tex]

Normal probability distribution

Problems of normally distributed distributions can be solved using the z-score formula.

In a set with mean [tex]\mu[/tex] and standard deviation [tex]\sigma[/tex], the z-score of a measure X is given by:

[tex]Z = \frac{X - \mu}{\sigma}[/tex]

The Z-score measures how many standard deviations the measure is from the mean. After finding the Z-score, we look at the z-score table and find the p-value associated with this z-score. This p-value is the probability that the value of the measure is smaller than X, that is, the percentile of X. Subtracting 1 by the p-value, we get the probability that the value of the measure is greater than X.

When we are approximating a binomial distribution to a normal one, we have that [tex]\mu = E(X)[/tex], [tex]\sigma = \sqrt{V(X)}[/tex].

About 12.5% of restaurant bills are incorrect.

This means that [tex]p = 0.125[/tex]

200 bills are selected at random

This means that [tex]n = 200[/tex]

Mean and standard deviation:

[tex]\mu = E(X) = np = 200*0.125 = 25[/tex]

[tex]\sigma = \sqrt{V(X)} = \sqrt{np(1-p)} = \sqrt{200*0.125*0.875} = 4.677[/tex]

Find the probability that at least 22 will contain an error.

Using continuity correction, this is [tex]P(X \geq 22 - 0.5) = P(X \geq 21.5)[/tex], which is 1 subtracted by the p-value of Z when X = 21.5. So

[tex]Z = \frac{X - \mu}{\sigma}[/tex]

[tex]Z = \frac{21.5 - 25}{4.677}[/tex]

[tex]Z = -0.75[/tex]

[tex]Z = -0.75[/tex] has a p-value of 0.2266.

1 - 0.2266 = 0.7734

0.7734 = 77.34% probability that at least 22 will contain an error. Probability above 50%, which means that this is likely to occur.

Practice Exercise 3.1 Fill in the blanks: (i) The factors of 12 are (ii) The least non-zero multiples of any number is (iii) ......... is a factor of every number. ing with Numbers​

Answers

Answer:

i.)1,2,3,4,6,12

ii).the number itself

iii.)1

What is the largest value of A according to the division operation given above?
A)300 B)314 C)400 D)450

Answers

Answer:

Hello,

Answer B

Step-by-step explanation:

Since A=15*20+B and B<15

The max for B is 14

==> 300+14=314

GIVING BRAINLIEST!!!!! AND ALL POINTS!!!!!!!!!!!!!!!!!!!
A right rectangular prism is packed with cubes of side length fraction 1 over 4 inch. If the prism is packed with 12 cubes along the length, 8 cubes along the width, and 5 cubes along the height, what is the volume of the prism?

fraction 2 and 3 over 4 cubic inches
fraction 3 and 3 over 4 cubic inches
fraction 7 and 1 over 4 cubic inches
fraction 7 and 1 over 2 cubic inches

Answers

Answer:

7 and 1 over 2 cubic inches ( 7 1/2 in³

Step-by-step explanation:

The height = 1/4 * 5 = 1 1/4 = 1.25

The width = 1/4 * 8 = 2

The length = 1/4 * 12 = 3

Volume = 1.25 * 2 * 3 = 2.5 * 3 = 7.5

0.5 is represented as 1/2

So answer : fraction 7 and 1 over 2 cubic inches or 7 1/2 in³

if my answer is incorrect, pls correct me!

If you like my answer and explanation, mark me as brainliest!

-Chetan K

37. The trip between 2 towns is exactly 90 miles. You have gone 40% of this distance. How far have
you gone?

Answers

Answer:

36 miles

Step-by-step explanation:

We want to find 40% of 90 miles

40% * 90

.40 * 90

36 miles

Distance=90mliesTravelled distance=40℅

We have to find travelled distance inorder to find this we have to find 40℅ of 90miles

[tex]\\ \Large\sf\longmapsto 90\times 40\℅[/tex]

[tex]\\ \Large\sf\longmapsto 90\times \dfrac{40}{100}[/tex]

[tex]\\ \Large\sf\longmapsto 9\times 4[/tex]

[tex]\\ \Large\sf\longmapsto 36miles [/tex]

10. What is the multiple zero and multiplicity of f(x) = (x - 3)(x - 3)(x + 5)?
Multiple zero is -3; multiplicity is 2
Multiple zero is 5; multiplicity is 1
Multiple zero is -5; multiplicity is 1
Multiple zero is 3; multiplicity is 2

Answers

Answer:

x=3, multiplicity of 2

x=-5, multiplicity of 1

Step-by-step explanation:

f(x) = (x - 3)(x - 3)(x + 5)

Rewriting

f(x) = (x - 3)^2(x + 5)

Setting equal to zero

0 =  (x - 3)^2(x + 5)

Using the zero product property

(x-3)^2  = 0   x+5 = 0

x-3 = 0         x= -5

x=3             x-5

Since x-3 was squared, the multiplicity is 2

Answer:

x=3, multiplicity of 2

x=-5, multiplicity of 1

Step-by-step explanation

Imagine that you are given two linear equations in slope-intercept form. You
notice that both the slopes and the y-intercepts are the same. How many
solutions would you expect for this system of equations?
O A. 1
ОВ. о
C. infinitely many
O D. cannot be determined
SURAT

Answers

Answer:

C. infinitely many

Step-by-step explanation:

If two equations in slope-intercept form have the same slope and y-intercept they must be the same line. Additionally, the solutions of a system of equations are wherever the two lines intersect. Since the lines are the same they must intersect at every point. Therefore, there are infinitely many solutions.

Other Questions
An adult ticket is twice the cost of a child's ticket. Angela paid $ 28 for two adult tickets and three children's tickets. Which of thefollowing equations could be used to find the price of a child's ticket?(A) x + 2x = 28(B) 3x + 2(2x) = 28(C)3(2x) + 2x = 28(D) 3x + 2x = 28 7x+2y=197x+8y=13Please solve it by Simultaneous method Responde falso o verdadero segn el caso ( ) Frau Frida empez a contar sus sueos a los siete aos ( ) Su hermano muri arrastrado por una Torrente ( ) La primera vez que Frau Frida se "alquil para soar" fue en su ciudad natal, en casa de un rentero. ( ) El narrador dej Viena despus de un sueo de Frau Frida what are the hospitals built by the European in Gold Coast A chocolate chip cookie manufacturing company recorded the number of chocolate chips in a sample of 60 cookies. The mean is 22.36 and the standard deviation is2.97 . Construct a 80% confidence interval estimate of the standard deviation of the numbers of chocolate chips in all such cookies. What is the atomic number of the atom in the diagram above please ? Outside a spherically symmetric charge distribution of net charge Q, Gauss's law can be used to show that the electric field at a given distance:___________.A) must be directed inward.B) acts like it originated in a point charge Q at the center of the distribution.C) must be directed outward.D) must be greater than zero.E) must be zero. Jay has a jar of beads. 6 purple beard, 8 blue beads, and 4 yellow beads. If he removes one bead at random, what is the probability that it will not be yellow? Find the equation of the line using the point-slope formula. Write the final equation using the slope-intercept form.perpendicular to7y = x 4and passes through the point(2, 1) Answer the question given above Which type of transformation is this? when a certain number is added to 11 and the result is multiplied by 3, the final result is 45 . find the number 4 glue sticks cost $7.76.Which equation would help determine the cost of 13 glue sticks? I need help can someone help me out Which of these is a question used to develop a thesis statement?A: Where can I find more information about my topic?B:What evidence supports my position?C:Why is this important to me or my reader?D:What does my reader believe about the topic? Please i need to find the era bounded by the following curves WILL GIVE BRAINLIEST AND 30 POINTS!Admission to a baseball game is $5.00 for general admission and $7.00 for reserved seats. The receipts were $6192.00 for 1136 paid admissions. How many of each ticket were sold? (Round to nearest integer if necessary.)_ general admissions tickets sold_ reserved seating tickets sold Story of the dog and 2 crow The scatterplot shows the number of bedrooms in a house and the selling price for that house.Calculate the residual for the house with 6 bedrooms, to the nearest thousand.The residual for the house with 6 bedrooms is ___-30k-26k26k30k If x=2 and y=1, what is the value of the expression 3(y - 1) + 2x ? a. 8 b. 10 c. 16 d. 18